Mathcenter Forum  

Go Back   Mathcenter Forum > คณิตศาสตร์โอลิมปิก และอุดมศึกษา > อสมการ
สมัครสมาชิก คู่มือการใช้ รายชื่อสมาชิก ปฏิทิน ข้อความวันนี้

ตั้งหัวข้อใหม่ Reply
 
เครื่องมือของหัวข้อ ค้นหาในหัวข้อนี้
  #1  
Old 26 ตุลาคม 2011, 09:54
PP_nine's Avatar
PP_nine PP_nine ไม่อยู่ในระบบ
กระบี่ประสานใจ
 
วันที่สมัครสมาชิก: 24 เมษายน 2010
ข้อความ: 607
PP_nine is on a distinguished road
Default โจทย์อสมการ 4-5 ข้อ

มีโจทย์ที่ดองไว้นานแล้ว เลยขุดเอามาถาม บางข้อเองยังทำไม่ได้เลย

1. จำนวนเต็ม $m_i \ge n_i \ge 0$ ทุก $i=1,2,...,k$ พิสูจน์ $$\pmatrix{m_1+m_2+...+m_k \\ n_1+n_2+...+n_k } \ge \prod_{i=1}^k \pmatrix{ m_i \\ n_i } $$

2. จำนวนจริง $a,b,c>0$ ซึ่ง $a+b+c=1$ พิสูจน์ $$\sum_{cyc} \frac{a}{\sqrt{b}(b+c)} \ge \frac{3}{2} \cdot \frac{1}{\sqrt{ab+bc+ca}}$$

3. $x_1,x_2,...x_n>0$ และ $x_1+x_2+...+x_n=1$ พิสูจน์ $$\sum_{i=1}^n \frac{x_i+n}{1+x_i^2} \le n^2$$ (Hint : $f(x)=\frac{x}{1+x^2}$ เป็นฟังก์ชันเพิ่มบน $[0,1]$)

4. $a,b,c>0$ และ $a^3+b^3+c^3=1$ พิสูจน์ $$a+b+c+\frac{1}{abc} \ge 3+\sqrt[3]{9}$$

5. $a,b,c>0$ และ $a+b+c \ge 6$ พิสูจน์ $$\sum_{cyc} \sqrt{a^2+\frac{1}{b+c}} \ge \frac{3 \sqrt{17}}{2}$$
__________________
keep your way.
ตอบพร้อมอ้างอิงข้อความนี้
  #2  
Old 13 พฤศจิกายน 2011, 18:01
AnDroMeDa's Avatar
AnDroMeDa AnDroMeDa ไม่อยู่ในระบบ
ลมปราณบริสุทธิ์
 
วันที่สมัครสมาชิก: 10 ตุลาคม 2011
ข้อความ: 114
AnDroMeDa is on a distinguished road
Default

อ้างอิง:
ข้อความเดิมเขียนโดยคุณ PP_nine View Post
2. จำนวนจริง $a,b,c>0$ ซึ่ง $a+b+c=1$ พิสูจน์ $$\sum_{cyc} \frac{a}{\sqrt{b}(b+c)} \ge \frac{3}{2} \cdot \frac{1}{\sqrt{ab+bc+ca}}$$

09 มีนาคม 2012 20:57 : ข้อความนี้ถูกแก้ไขแล้ว 2 ครั้ง, ครั้งล่าสุดโดยคุณ AnDroMeDa
ตอบพร้อมอ้างอิงข้อความนี้
  #3  
Old 13 พฤศจิกายน 2011, 18:47
AnDroMeDa's Avatar
AnDroMeDa AnDroMeDa ไม่อยู่ในระบบ
ลมปราณบริสุทธิ์
 
วันที่สมัครสมาชิก: 10 ตุลาคม 2011
ข้อความ: 114
AnDroMeDa is on a distinguished road
Default

อ้างอิง:
ข้อความเดิมเขียนโดยคุณ PP_nine View Post
3. $x_1,x_2,...x_n>0$ และ $x_1+x_2+...+x_n=1$ พิสูจน์ $$\sum_{i=1}^n \frac{x_i+n}{1+x_i^2} \le n^2$$ (Hint : $f(x)=\frac{x}{1+x^2}$ เป็นฟังก์ชันเพิ่มบน $[0,1]$)
ตอบพร้อมอ้างอิงข้อความนี้
  #4  
Old 13 พฤศจิกายน 2011, 19:28
BLACK-Dragon's Avatar
BLACK-Dragon BLACK-Dragon ไม่อยู่ในระบบ
กระบี่ประสานใจ
 
วันที่สมัครสมาชิก: 04 พฤศจิกายน 2010
ข้อความ: 719
BLACK-Dragon is on a distinguished road
Default

อ้างอิง:
ข้อความเดิมเขียนโดยคุณ PP_nine View Post
เป็นฟังก์ชันเพิ่มบน $[0,1]$)

4. $a,b,c>0$ และ $a^3+b^3+c^3=1$ พิสูจน์ $$a+b+c+\frac{1}{abc} \ge 3+\sqrt[3]{9}$$
อยากทราบนิดนึงนะครับ มันเป็นน้อยกว่าเท่ากับหรือเปล่าเอ่ย

ส่วนข้อ 5 แทน $a=b=c=3$ แล้วมันไม่จริงน่ะครับ

13 พฤศจิกายน 2011 19:57 : ข้อความนี้ถูกแก้ไขแล้ว 1 ครั้ง, ครั้งล่าสุดโดยคุณ BLACK-Dragon
ตอบพร้อมอ้างอิงข้อความนี้
  #5  
Old 14 พฤศจิกายน 2011, 00:43
AnDroMeDa's Avatar
AnDroMeDa AnDroMeDa ไม่อยู่ในระบบ
ลมปราณบริสุทธิ์
 
วันที่สมัครสมาชิก: 10 ตุลาคม 2011
ข้อความ: 114
AnDroMeDa is on a distinguished road
Default

อ้างอิง:
ข้อความเดิมเขียนโดยคุณ PP_nine View Post
5. $a,b,c>0$ และ $a+b+c \ge 6$ พิสูจน์ $$\sum_{cyc} \sqrt{a^2+\frac{1}{b+c}} \ge \frac{3 \sqrt{17}}{2}$$
ข้อนี้ทำได้แต่กรณี $a+b+c=6$ แต่ $a+b+c>6$ ทำไม่เป็นอ่ะครับ- -*

#4 อย่าลืมดู$a^3+b^3+c^3=1$
ตอบพร้อมอ้างอิงข้อความนี้
  #6  
Old 14 พฤศจิกายน 2011, 13:20
จูกัดเหลียง's Avatar
จูกัดเหลียง จูกัดเหลียง ไม่อยู่ในระบบ
ลมปราณไร้สภาพ
 
วันที่สมัครสมาชิก: 21 กุมภาพันธ์ 2011
ข้อความ: 1,234
จูกัดเหลียง is on a distinguished road
Default

#5 ทำไงครับ (Jensen's ???)
__________________
Vouloir c'est pouvoir

14 พฤศจิกายน 2011 13:25 : ข้อความนี้ถูกแก้ไขแล้ว 1 ครั้ง, ครั้งล่าสุดโดยคุณ จูกัดเหลียง
ตอบพร้อมอ้างอิงข้อความนี้
  #7  
Old 14 พฤศจิกายน 2011, 14:35
BLACK-Dragon's Avatar
BLACK-Dragon BLACK-Dragon ไม่อยู่ในระบบ
กระบี่ประสานใจ
 
วันที่สมัครสมาชิก: 04 พฤศจิกายน 2010
ข้อความ: 719
BLACK-Dragon is on a distinguished road
Default

อ้างอิง:
ข้อความเดิมเขียนโดยคุณ PP_nine View Post

5. $a,b,c>0$ และ $a+b+c \ge 6$ พิสูจน์ $$\sum_{cyc} \sqrt{a^2+\frac{1}{b+c}} \ge \frac{3 \sqrt{17}}{2}$$
ถ้าเป็น $a+b+c=6$ อย่างที่คุณ AnDroMeDa นะครับ

$$\displaystyle \sum_{cyc} \sqrt{a^2+\dfrac{1}{b+c}} = \sum_{cyc} \sqrt{\underbrace{\dfrac{a^2}{16}+\dfrac{a^2}{16}+...+\dfrac{a^2}{16}}_{16} +\dfrac{1}{b+c}} \geq \sum_{cyc} \dfrac{1}{\sqrt{17}}\left(\,4a+\dfrac{1}{\sqrt{b+c}}\right) $$
$$\displaystyle \geq \dfrac{1}{\sqrt{17}}\left(\,24+\dfrac{3}{\sqrt[6]{(a+b)(b+c)(c+a)}}\right) \geq \dfrac{1}{\sqrt{17}} \left(\,24+\dfrac{3}{\sqrt{\dfrac{2(a+b+c)}{3}}}\right) =\dfrac{3\sqrt{17}}{2}$$
ตอบพร้อมอ้างอิงข้อความนี้
  #8  
Old 14 พฤศจิกายน 2011, 17:08
PP_nine's Avatar
PP_nine PP_nine ไม่อยู่ในระบบ
กระบี่ประสานใจ
 
วันที่สมัครสมาชิก: 24 เมษายน 2010
ข้อความ: 607
PP_nine is on a distinguished road
Default

#4 ถ้า $a=b=c=3$ ผมกดเครื่องคิดเลขแล้วมันก็ยังจริงนี่ครับ

อ้างอิง:
ข้อความเดิมเขียนโดยคุณ PP_nine View Post
5. $a,b,c>0$ และ $a+b+c \ge 6$ พิสูจน์ $$\sum_{cyc} \sqrt{a^2+\frac{1}{b+c}} \ge \frac{3 \sqrt{17}}{2}$$
ข้อนี่ใช้ lemma ตัวนึงที่น่าจะเคยเห็นกันบ้างแล้ว ผมจะเว้นพิสูจน์ไว้ ลองเอาไปทำดูนะครับ

สำหรับ $x_1,x_2,...,x_n,y_1,y_2,...,y_n \in \mathbb{R} ^+$ $$\sqrt{x_1^2+y_1^2}+\sqrt{x_2^2+y_2^2}+...+\sqrt{x_n^2+y_n^2} \ge \sqrt{(x_1+x_2+...+x_n)^2+(y_1+y_2+...+y_n)^2}$$ สมการเกิดเมื่ออัตราส่วน $x_i/y_i$ เท่ากันทุก $i=1,2,...,n$

apply lemma กับ LHS ได้ว่า $$\sum_{cyc} \sqrt{a^2+\left(\, \frac{1}{\sqrt{b+c}}\right) ^2} \ge \sqrt{(a+b+c)^2+\left(\,\frac{1}{\sqrt{a+b}}+\frac{1}{\sqrt{b+c}}+\frac{1}{\sqrt{c+a}}\right)^2 }$$ โดย AM-HM และ Cauchy-Schwarz ได้ $$\left(\,\frac{1}{\sqrt{a+b}}+\frac{1}{\sqrt{b+c}}+\frac{1}{\sqrt{c+a}}\right)^2 \ge \left(\, \frac{9}{\sqrt{a+b}+\sqrt{b+c}+\sqrt{c+a}}\right) ^2 \ge \frac{81}{3(2a+2b+2c)}$$ อสมการเดิมจึงได้เป็น $$\sum_{cyc} \sqrt{a^2+\left(\, \frac{1}{\sqrt{b+c}}\right) ^2} \ge \sqrt{(a+b+c)^2+\frac{27}{2(a+b+c)}}$$ $$\sum_{cyc} \sqrt{a^2+\left(\, \frac{1}{\sqrt{b+c}}\right) ^2} \ge \sqrt{(a+b+c)^2+\frac{216}{a+b+c}-\frac{405}{2(a+b+c)}}$$ ใช้ AM-GM กับสองก้อนแรกใน RHS และใช้อสมการ $a+b+c \ge 6$ กับก้อนสุดท้ายใน RHS ได้เป็น $$\sum_{cyc} \sqrt{a^2+\left(\, \frac{1}{\sqrt{b+c}}\right) ^2} \ge \sqrt{2\sqrt{216(a+b+c)}-\frac{405}{2 \cdot 6}}$$ $$\sum_{cyc} \sqrt{a^2+\left(\, \frac{1}{\sqrt{b+c}}\right) ^2} \ge \sqrt{2\sqrt{216(6)}-\frac{405}{2 \cdot 6}}$$ จัดรูปจนได้ $$\sum_{cyc} \sqrt{a^2+\left(\, \frac{1}{\sqrt{b+c}}\right) ^2} \ge \frac{3\sqrt{17}}{2}$$ สมการเกิดเมื่อ $a=b=c=2$ เท่านั้น
__________________
keep your way.

14 พฤศจิกายน 2011 17:09 : ข้อความนี้ถูกแก้ไขแล้ว 1 ครั้ง, ครั้งล่าสุดโดยคุณ PP_nine
ตอบพร้อมอ้างอิงข้อความนี้
  #9  
Old 14 พฤศจิกายน 2011, 17:25
AnDroMeDa's Avatar
AnDroMeDa AnDroMeDa ไม่อยู่ในระบบ
ลมปราณบริสุทธิ์
 
วันที่สมัครสมาชิก: 10 ตุลาคม 2011
ข้อความ: 114
AnDroMeDa is on a distinguished road
Default

โอ้ให้เป็น Best Solutionเลยครับ555(ผมก็ใช้มินคอฟสกีแต่ไปไม่ถูก)

14 พฤศจิกายน 2011 17:26 : ข้อความนี้ถูกแก้ไขแล้ว 1 ครั้ง, ครั้งล่าสุดโดยคุณ AnDroMeDa
ตอบพร้อมอ้างอิงข้อความนี้
  #10  
Old 14 พฤศจิกายน 2011, 17:26
BLACK-Dragon's Avatar
BLACK-Dragon BLACK-Dragon ไม่อยู่ในระบบ
กระบี่ประสานใจ
 
วันที่สมัครสมาชิก: 04 พฤศจิกายน 2010
ข้อความ: 719
BLACK-Dragon is on a distinguished road
Default

อ้างอิง:
ข้อความเดิมเขียนโดยคุณ PP_nine View Post
#4 ถ้า $a=b=c=3$ ผมกดเครื่องคิดเลขแล้วมันก็ยังจริงนี่ครับ



ข้อนี่ใช้ lemma ตัวนึงที่น่าจะเคยเห็นกันบ้างแล้ว ผมจะเว้นพิสูจน์ไว้ ลองเอาไปทำดูนะครับ

สำหรับ $x_1,x_2,...,x_n,y_1,y_2,...,y_n \in \mathbb{R} ^+$ $$\sqrt{x_1^2+y_1^2}+\sqrt{x_2^2+y_2^2}+...+\sqrt{x_n^2+y_n^2} \ge \sqrt{(x_1+x_2+...+x_n)^2+(y_1+y_2+...+y_n)^2}$$ สมการเกิดเมื่ออัตราส่วน $x_i/y_i$ เท่ากันทุก $i=1,2,...,n$

apply lemma กับ LHS ได้ว่า $$\sum_{cyc} \sqrt{a^2+\left(\, \frac{1}{\sqrt{b+c}}\right) ^2} \ge \sqrt{(a+b+c)^2+\left(\,\frac{1}{\sqrt{a+b}}+\frac{1}{\sqrt{b+c}}+\frac{1}{\sqrt{c+a}}\right)^2 }$$ โดย AM-HM และ Cauchy-Schwarz ได้ $$\left(\,\frac{1}{\sqrt{a+b}}+\frac{1}{\sqrt{b+c}}+\frac{1}{\sqrt{c+a}}\right)^2 \ge \left(\, \frac{9}{\sqrt{a+b}+\sqrt{b+c}+\sqrt{c+a}}\right) ^2 \ge \frac{81}{3(2a+2b+2c)}$$ อสมการเดิมจึงได้เป็น $$\sum_{cyc} \sqrt{a^2+\left(\, \frac{1}{\sqrt{b+c}}\right) ^2} \ge \sqrt{(a+b+c)^2+\frac{27}{2(a+b+c)}}$$ $$\sum_{cyc} \sqrt{a^2+\left(\, \frac{1}{\sqrt{b+c}}\right) ^2} \ge \sqrt{(a+b+c)^2+\frac{216}{a+b+c}-\frac{405}{2(a+b+c)}}$$ ใช้ AM-GM กับสองก้อนแรกใน RHS และใช้อสมการ $a+b+c \ge 6$ กับก้อนสุดท้ายใน RHS ได้เป็น $$\sum_{cyc} \sqrt{a^2+\left(\, \frac{1}{\sqrt{b+c}}\right) ^2} \ge \sqrt{2\sqrt{216(a+b+c)}-\frac{405}{2 \cdot 6}}$$ $$\sum_{cyc} \sqrt{a^2+\left(\, \frac{1}{\sqrt{b+c}}\right) ^2} \ge \sqrt{2\sqrt{216(6)}-\frac{405}{2 \cdot 6}}$$ จัดรูปจนได้ $$\sum_{cyc} \sqrt{a^2+\left(\, \frac{1}{\sqrt{b+c}}\right) ^2} \ge \frac{3\sqrt{17}}{2}$$ สมการเกิดเมื่อ $a=b=c=2$ เท่านั้น
คาราวะ 10 จอกเลยครับ
ตอบพร้อมอ้างอิงข้อความนี้
  #11  
Old 14 พฤศจิกายน 2011, 17:26
PP_nine's Avatar
PP_nine PP_nine ไม่อยู่ในระบบ
กระบี่ประสานใจ
 
วันที่สมัครสมาชิก: 24 เมษายน 2010
ข้อความ: 607
PP_nine is on a distinguished road
Default

#4 ส่วนของข้อ 4 ถูกแล้วครับ เพิ่งทำออกพอดี

อ้างอิง:
ข้อความเดิมเขียนโดยคุณ PP_nine View Post
4. $a,b,c>0$ และ $a^3+b^3+c^3=1$ พิสูจน์ $$a+b+c+\frac{1}{abc} \ge 3+\sqrt[3]{9}$$
จาก $a^3+b^3+c^3=1$ โดย Power mean และ AM-GM ได้ $a+b+c \le \sqrt[3]{9}$ และ $abc \le \frac{(a+b+c)^3}{27} \le \frac{1}{3}$

ดังนั้น $$(a+b+c)+\frac{1}{abc} \ge \frac{(a+b+c)^3}{3\sqrt[3]{3}}+\frac{1}{abc}$$ แต่ $$ \frac{(a+b+c)^3}{3\sqrt[3]{3}}+\frac{1}{abc}= \frac{(a+b+c)^3}{3\sqrt[3]{3}}+\frac{1}{\sqrt[3]{3}abc}+\left(\,1-\frac{1}{\sqrt[3]{3}}\right) \frac{1}{abc}$$ ดังนั้น $$(a+b+c)+\frac{1}{abc} \ge \frac{(a+b+c)^3}{3\sqrt[3]{3}}+\frac{1}{\sqrt[3]{3}abc}+\left(\,1-\frac{1}{\sqrt[3]{3}}\right) \frac{1}{abc}$$ โดยอสมการที่เก็บไว้ตอนแรก จะได้ $$(a+b+c)+\frac{1}{abc} \ge \frac{(a+b+c)^3}{3\sqrt[3]{3}}+\frac{27}{\sqrt[3]{3}(a+b+c)^3}+\left(\,1-\frac{1}{\sqrt[3]{3}}\right) (3)$$ และใช้ AM-GM กับสองก้อนแรกใน RHS ได้ $$a+b+c+\frac{1}{abc} \ge 2 \sqrt{\frac{9}{\sqrt[3]{9}}}+(3-\sqrt[3]{9})$$ $$a+b+c+\frac{1}{abc} \ge 2 \frac{3}{\sqrt[3]{3}}+3-\sqrt[3]{9} $$ $$a+b+c+\frac{1}{abc} \ge 2\sqrt[3]{9}+3-\sqrt[3]{9}=3+\sqrt[3]{9}$$ สมการเกิดเมื่อ $a=b=c=\frac{1}{\sqrt[3]{3}}$ เท่านั้น
__________________
keep your way.

14 พฤศจิกายน 2011 17:34 : ข้อความนี้ถูกแก้ไขแล้ว 1 ครั้ง, ครั้งล่าสุดโดยคุณ PP_nine
ตอบพร้อมอ้างอิงข้อความนี้
  #12  
Old 14 พฤศจิกายน 2011, 17:34
AnDroMeDa's Avatar
AnDroMeDa AnDroMeDa ไม่อยู่ในระบบ
ลมปราณบริสุทธิ์
 
วันที่สมัครสมาชิก: 10 ตุลาคม 2011
ข้อความ: 114
AnDroMeDa is on a distinguished road
Default

เยี่ยมมากเลยครับฝันมาได้ไงนี่
$$ \frac{(a+b+c)^3}{3\sqrt[3]{3}}+\frac{1}{abc}= \frac{(a+b+c)^3}{3\sqrt[3]{3}}+\frac{1}{\sqrt[3]{3}abc}+\left(\,1-\frac{1}{\sqrt[3]{3}}\right) \frac{1}{abc}$$
ตอบพร้อมอ้างอิงข้อความนี้
  #13  
Old 14 พฤศจิกายน 2011, 17:38
PP_nine's Avatar
PP_nine PP_nine ไม่อยู่ในระบบ
กระบี่ประสานใจ
 
วันที่สมัครสมาชิก: 24 เมษายน 2010
ข้อความ: 607
PP_nine is on a distinguished road
Default

ก็ลองแทนค่า $a=b=c$ ดูครับ แล้วจากนั้นก็ทดตัวเลขให้มันเท่ากันเวลาใช้ AM-GM ที่เหลือก็มีแค่เก็บรายละเอียดจุกจิกให้เป็นตัวเลขให้หมด

ทีนี้ก็เหลือข้อ 1 ที่ยังไม่รู้จะเริ่มยังไงเลย
__________________
keep your way.
ตอบพร้อมอ้างอิงข้อความนี้
  #14  
Old 14 พฤศจิกายน 2011, 17:42
AnDroMeDa's Avatar
AnDroMeDa AnDroMeDa ไม่อยู่ในระบบ
ลมปราณบริสุทธิ์
 
วันที่สมัครสมาชิก: 10 ตุลาคม 2011
ข้อความ: 114
AnDroMeDa is on a distinguished road
Default

ข้อ 1 Jensen จะไปต่อได้ไหมอ่ะครับ
ตอบพร้อมอ้างอิงข้อความนี้
  #15  
Old 14 พฤศจิกายน 2011, 19:22
Thgx0312555's Avatar
Thgx0312555 Thgx0312555 ไม่อยู่ในระบบ
กระบี่ประสานใจ
 
วันที่สมัครสมาชิก: 12 สิงหาคม 2011
ข้อความ: 885
Thgx0312555 is on a distinguished road
Default

ขออนุญาตเริ่มข้อ 1 ครับ (ติดตอนท้าย)
จะสามารถพิสูจน์ได้อย่างง่ายว่า $A \geqslant B$ เช่น $(a+b)! \geqslant a!b!$ ลองพิสูจน์ดูครับ

__________________
----/---~Alice~ จงรับรู้ไว้ ชื่อแห่งสีสันหนึ่งเดียวที่แสดงผล
---/---- ~Blue~ นี่คือ สีแห่งความหลังอันกว้างใหญ่ของเว็บบอร์ดนี้

14 พฤศจิกายน 2011 23:17 : ข้อความนี้ถูกแก้ไขแล้ว 4 ครั้ง, ครั้งล่าสุดโดยคุณ Thgx0312555
เหตุผล: คิดผิด
ตอบพร้อมอ้างอิงข้อความนี้
ตั้งหัวข้อใหม่ Reply



กฎการส่งข้อความ
คุณ ไม่สามารถ ตั้งหัวข้อใหม่ได้
คุณ ไม่สามารถ ตอบหัวข้อได้
คุณ ไม่สามารถ แนบไฟล์และเอกสารได้
คุณ ไม่สามารถ แก้ไขข้อความของคุณเองได้

vB code is On
Smilies are On
[IMG] code is On
HTML code is Off
ทางลัดสู่ห้อง


เวลาที่แสดงทั้งหมด เป็นเวลาที่ประเทศไทย (GMT +7) ขณะนี้เป็นเวลา 05:26


Powered by vBulletin® Copyright ©2000 - 2024, Jelsoft Enterprises Ltd.
Modified by Jetsada Karnpracha